Approximating the Sum of a Series to Within a Given Error Bound

  • Thread starter Thread starter APolaris
  • Start date Start date
  • Tags Tags
    Series
Click For Summary
To estimate the sum of the series from 2 to infinity of 1 / (n^2 + 4) within an error of 0.1, the integral from n to infinity of 1 / (x^2 + 4) is utilized. The inequalities involving the remainder of the series lead to determining the appropriate value of n by solving for when the difference between the integrals is less than 0.2. Specifically, the condition (π/4 - 1/2 arctan[(n+1)/2]) < 0.1 provides a lower bound for n, resulting in n being approximately 8.87. Once n is established, the series sum can be approximated using the average of the two integral values, ensuring the error remains within the specified limit. This method effectively narrows down the number of terms needed for a precise approximation of the series sum.
APolaris
Messages
8
Reaction score
0
The question is: estimate the sum, from 2 to infinity, of 1 / (n^2 + 4) to within 0.1 of exact value.

I have the following: the integral from n to inf. of 1 / (x^2 + 4) is (pi/4 - 1/2 arctan(n/2)).

Next, in order to find the number of partial sums to use, set Sn + the integral from n+1 to infinity < S < Sn + the integral from n to infinity.

This gives (pi/4 - 1/2 arctan[(n+1)/2]) < (remainder) < (pi/4 - 1/2 arctan(n/2)).

I can only assume from here that you are meant to subtract the lesser from the greater and set (1/2 arctan[(n+1)/2]) - 1/2 arctan(n/2) < 0.2. So arctan((n+1)/2) - arctan(n/2) < 0.4. What?
 
Last edited:
Physics news on Phys.org
The error of the integral approximation will be between the two integrals you have, so you want
\int_{n+1}^\infty \frac{dx}{x^2 + 4} \leq 0.1 \leq \int_n^\infty \frac{dx}{x^2 + 4}
These two separate inequalities will tell you what range of values of n you should choose to approximate the series within an error bound of 0.1. For this problem, it turns out there is only one integer between these two integrals.
\frac{\pi}{4} - \frac{1}{2}\arctan\left(\frac{n + 1}{2}\right) \leq 0.1
for example gives us the lower bound when solved of
n \geq 2\tan\left(\frac{\pi}{2}-0.2\right)-1 \approx 8.86631
Once you have the value of n, you can use it in the inequality
s_n + \int_{n+1}^\infty \frac{dx}{x^2 + 4} \leq s \leq s_n + \int_n^\infty \frac{dx}{x^2 + 4}
Adding the two values and dividing by 2 does not change the error of the value, so you can approximate the value s by taking the average of the two values outside.
 
Last edited:
Question: A clock's minute hand has length 4 and its hour hand has length 3. What is the distance between the tips at the moment when it is increasing most rapidly?(Putnam Exam Question) Answer: Making assumption that both the hands moves at constant angular velocities, the answer is ## \sqrt{7} .## But don't you think this assumption is somewhat doubtful and wrong?

Similar threads

  • · Replies 1 ·
Replies
1
Views
2K
  • · Replies 3 ·
Replies
3
Views
2K
  • · Replies 16 ·
Replies
16
Views
2K
  • · Replies 7 ·
Replies
7
Views
1K
Replies
3
Views
2K
  • · Replies 1 ·
Replies
1
Views
2K
  • · Replies 7 ·
Replies
7
Views
2K
Replies
8
Views
2K
Replies
2
Views
1K
Replies
10
Views
2K